VIQ - TEST PAPER 5 Flashcards

1
Q

@# 3.A 2-year old boy with proptosis and cats eye was investigated with CT for persistentheadache. Axial images showed a densely calcified mass replacing the right eyeball. The opticnerve was also calcified and surrounded by tumour, which had replaced most of theperiorbital fat. The optic canal was expanded with extension of the mass in the middle cranialfossa. What is the diagnosis?
A. Malignant melanoma of the choroid
B. Rhabdomyosarcoma
C. Coats’ disease
D. Neuroblastoma metastasis
E. Retinoblastoma

A

3.E. Retinoblastoma

Retinoblastoma is the most common tumour of the globe in children. It is seen in children lessthan 3 years, presenting with leukocoria; 75% are unilateral and unifocal, 25% are bilateral or unilateral multifocal. When seen bilaterally in conjunction with pineoblastoma, it is called trilateral retinoblastoma.
CT is preferred and shows clumped or punctate calcification (95%) in the posterior aspect of the eye, extending into the vitreous humor with minimal enhancement. Absence of calcification makes retinoblastoma unlikely. On MRI, retinoblastomas are hyperintense on T1 -weighted and hypointense on T2-weighted images, possibly due to calcification or paramagnetic tumour protein. MRI is belter at depicting tumour extension along optic nerves and intracranially. CT is better at showing bone dest ruction including expansion of the optic canal

How well did you know this?
1
Not at all
2
3
4
5
Perfectly
2
Q

@# 5. With regard to stress fractures affecting the lower limb in athletes, all of the following statements are correct, except
A. Anterior tibial stress fractures have a higher propensity of non-union.
B. Fibular stress fractures affect the proximal end.
C. Femoral stress fractures can affect the neck or shaft.
D. ‘Female athlete triad’ is associated with femoral and sacral fractures.
E. Tibial stress fracture can be transverse or longitudinal.

A
  1. B. Fibular stress fractures affect the proximal end.

The tibia is the most commonly involved bone, accounting for up to almost half of stress fractures reported in some series.

Two types of tibial stress fractures have been described - transverse and longitudinal. Transverse fractures are more common and can occur on the compression side (posterior) or tension side (anterior).

Posterior transverse fractures of the tibial shaft are most commonly seen in long-distance runners.

Tension stress transverse fractures of the anterior tibial shaft occur more commonly in jumpers and have a higher propensity for non-union and progression to an acute complete fracture.

The most coimnon site of fibular stress fracture in runners is the lower fibula, just proximal to the tibiotalar syndesmosis.

Femoral neck stress fractures are associated with the classic ‘female athlete triad’ of amenorrhoea, osteoporosis and eating disorders.

Femoral neck stress fractures are often classified as tension side (superolateral or transverse) or compression side (inferolateral), with the contention that tension side fractures are associated with poorer prognosis and are potentially unstable.

The femoral shaft is particularly susceptible to repetitive stresses on the medial compression side of the femur at the junction of the proximal and middle thirds.
Fractures of the sacrum have predominantly been described in long-distance runners, particularly women, but have also been reported in hockey players. Sacral stress fractures are also associated with the female athlete triad.

How well did you know this?
1
Not at all
2
3
4
5
Perfectly
3
Q

@# 6. A 3-year-old girl is referred to an endocrine clinic with unilateral jaw swelling noted at the dentist. Her general practitioner has also reported that she has signs of precocious puberty. An X-ray of the facial bones demonstrates expansion of the frontal bone and right side of the mandible. She is likely to have which other associated condition?
A. Neurofibromatosis
B. Madelung deformity
C. Lisch nodule
D. Hyperthyroidism
E. Hypothyroidism

A
  1. D. Hyperthyroidism

The child has McCune-Albright syndrome (MAS), which is defined as the association of polyostotic fibrous dysplasia (PFD), precocious puberty, cafe au lait spots and other endocrinopathies caused by the hyperactivity of various endocrine glands. Among the endocrine syndromes described in association with MAS are (1) hyperthyroidism, (2) acromegaly,
(3) gonadotrophinomas, (4) hyperprolactinaemia, (5) Cushing syndrome,
(6) hyperparathyroidism, (7) gynaecomastia and (8) hypophosphataemic rickets.
Lisch nodules are associated with neurofibromatosis. Fibrous dysplasia in MAS can involve any bone but most commonly affects the long bones, ribs, skull and facial bones. There is no association between Madelung deformity and MAS.

How well did you know this?
1
Not at all
2
3
4
5
Perfectly
4
Q

@# 7. A 35-year-old man known to the ENT for sinonasal disease presents to the chest clinic with productive cough. The house officer examining the patient struggled to hear the heart sounds properly. HRCT, among other findings, shows left lower lobe bronchiectasis without any central endobronchial mass to explain the focal bronchiectasis. What is the
likely diagnosis?
A. Cystic fibrosis (CF)
B. Sarcoidosis
C. William-Campbell syndrome
D. Allergic bronchopulmonary aspergillosis (ABPA)
E. Kartagener’s syndrome

A
  1. E. Kartagener’s syndrome

Kartagener’s syndrome refers to the clinical combination of situs inversus, chronic sinusitis and bronchiectasis in a subset of patients with ciliary dyskinesia.
Patients with primary ciliary dyskinesia typically have varicoid bronchiectasis preferentially affecting the lower lungs, particularly the right middle lobe and lingula, with chronic volume loss and consolidation. Tree-in-bud nodules related to infection and mucous plugging secondary to impaired clearance are also frequently seen. The associated finding of dextrocardia can be seen at chest radiography in cases of Kartagener’s syndrome.

How well did you know this?
1
Not at all
2
3
4
5
Perfectly
5
Q

@# 8. A 56 year old man with gallstone pancreatitis is referred for a CT of the abdomen. The CT shows multiple cysts around the tail of the pancreas with further cysts in the lesser sac and left paracolic gutter. These are reported as pseudocysts. Which one of the following statements regarding pancreatic pseudocysts is false?
A. They usually take 4-6 weeks to mature.
B. They can be multiple.
C. They have an epithelial lining.
D. They may communicate with the pancreatic ductal system.
E. They may be extrapancreatic in location

A
  1. C. They have an epithelial lining.

Overall, pseudocysts are the most common cystic lesions of the pancreas. These lesions occur in the setting of pancreatitis, resulting from haemorrhagic fat necrosis and encapsulation of pancreatic secretions by granulation tissue and a fibrous capsule. The MR imaging appearance of pseudocysts may evolve over time; they are often irregularly marginated early in their formation but become well circumscribed, with a thickened enhancing wall, over a period of several weeks. Blood products and necrotic or proteinaceous debris are commonly present and produce intrinsically increased Tl signal intensity. The thickened and enhancing cyst wall seen on images corresponds to a thick rim of granulation tissue and fibrosis that is uniformly seen at histologic analysis. Other changes of acute or chronic pancreatitis are frequently seen in association with pseudocysts, and MR imaging may be the imaging modality of choice for depicting the features of parenchymal pancreatic disease.
MR imaging has proved superior to CT for demonstrating internal complexity in pseudocysts. Furthermore, the signal intensity increase in tissues surrounding a complicated pseudocyst on T2-weighted fat-suppressed images correlates with the degree of inflammation present. However, in patients with a pseudocyst, the cause of inflammation is more likely to be chemical irritation than infection, and it may be impossible to differentiate between an infectious process and other possible causes on the basis of imaging features alone. Clinical manifestations maybe similarly unhelpful, since the symptoms of chemical irritation may be identical to those of sepsis. Moreover, pancreatic pseudocysts may dissect along abdominopelvic fascial planes to sites remote from the pancreas (e.g., liver, pleura or mediastinum).
Fistulation may occur between a pseudocyst and one or more vascular structures.

How well did you know this?
1
Not at all
2
3
4
5
Perfectly
6
Q

@# 10. A 52-year-old woman presents to the orthopaedic outpatient clinic with a painful forefoot. On examination, there was a painful response elicited by Mulder’s manoeuvre. Which one of the following statements concerning Morton neuromas m the forefoot is false?
A. They are often seen in young and middle-aged women.
B. The inter space between the third and fourth toes is the most commonly affected site.
C. The characteristic MR finding is a nodule with low signal intensity on T1W images.
D. Ultrasound is a sensitive modality in identifying the lesion.
E. Gradient echo MR sequences elicit blooming artefact in the lesion.

A
  1. E. Gradient echo MR sequences elicit blooming artefact in the lesion.

Morton neuromas are masses composed of interdigital perineural fibrosis and nerve
degeneration. Morton neuroma occurs between the metatarsal heads, most commonly between the third and fourth toes. Morton neuroma is more common in women, and high-heeled shoes have been implicated as a causative factor. Pain at the metatarsal head, often radiating to the toes, is characteristic.
The MRI appearance is that of a tear-drop-shaped soft-tissue mass between the metatarsal heads, projecting inferiorly into the plantar subcutaneous fat and located plantar to the intermetatarsal ligament. The mass is typically intermediate in signal intensity on T1-weighted images. It is iso- or hypointense relative to fat on T2-weighted images, resulting in poor lesion conspicuity. The use of gadopentetate dimeglumine is helpful because intense enhancement typically occurs on fat suppressed T1-weighted images, increasing the conspicuity of the lesion

How well did you know this?
1
Not at all
2
3
4
5
Perfectly
7
Q

@# 13. A 77-year-old man with chronic inflammatory disease and renal failure is known to have secondary amyloidosis. All of the following are features of amyloid involvement of the respiratory system, except
A. Interstitial septal thickening
B. Cavitating nodules
C. Focal amyloidoma
D. Calcification of central airways
E. Calcification in peripheral consolidation

A
  1. B. Cavitating nodules

Amyloidosis refers to a group of disorders characterised by the deposition of abnormal protein material in extracellular tissue.
Tracheobronchial amyloidosis generally presents with symptoms of airway obstruction. Classic radiological signs include nodular and irregular narrowing of the tracheal lumen, airway wall thickening and calcified amyloid deposits. Lobar or segmental collapse may be seen.
Pulmonary involvement by amyloid may be localised or diffuse. Radiologically, the diffuse parenchymal and alveolar septal forms of amyloid deposits appear as non-specific diffuse interstitial or alveolar opacities. HRCT reveals interlobular septal thickening with a predominant basilar and peripheral distribution, small well-defined nodules (2-4 mm) and confluent
consolidations located predominantly in the subpleural regions. Some nodules may show calcifications.
Nodular amyloid deposits appear in multiple sites; focal deposits are less common. Amyloid nodules are generally in the lower lobes and peripheral and subpleural areas. They are sharply defined with lobulated contours, contain calcification (in about 50%), are of multiple shapes and sizes and grow slowly with no regression. Cavitation is very rare.

How well did you know this?
1
Not at all
2
3
4
5
Perfectly
8
Q

@# 14. An US of the abdomen in a 43-year-old woman with a known underlying chronic condition demonstrates small cysts in the pancreas. She is sent for a dual phase CT for further characterisation. The scan shows multiple true cysts with no obvious suspicious features. Which one of the following conditions is associated with true pancreatic cysts?
A. Tuberous sclerosis
B. Von Hippel-Lindau disease
C. Neurofibromatosis Type 1
D. Autosomal recessive polycystic kidney disease
E. Multiple neuroendocrine neoplasia

A
  1. B. Von Hippel-Lindau disease

Pancreatic involvement in VHL disease includes simple pancreatic cysts (50%-91%), serous microcystic adenomas (12%) and rarely adenocarcinomas. Pancreatic neuroendocrine tumours (5%-17%) also occur. Combined lesions occur, but neuroendocrine tumours and cystic lesions only rarely exist together. The reported prevalence of pancreatic involvement in VHL disease varies from 0% in some family groups to 77% in others.
Pancreatic cysts are extremely rare in the general population; therefore, the presence of a single cyst in an individual undergoing VHL disease screening because of a family history makes it highly likely that the person has VHL disease. In general, cystic pancreatic lesions in VHL disease are asymptomatic or associated with only mild symptoms. As a result, they are typically detected during screening examinations and may therefore facilitate the identification of gene carriers. In addition, pancreatic lesions may be the only abdominal manifestation and may precede any other manifestation by several years; thus, recognition permits earlier diagnosis of VHL disease.

How well did you know this?
1
Not at all
2
3
4
5
Perfectly
9
Q

@# 15. A woman who is 22 weeks pregnant is referred for routine anomaly scan to the US department. US shows that the ventricular atrium measure 14 mm at the level of the posterior margin of the glomus of the choroids plexus on an axial plain through the level of the thalami. What is the next appropriate step?
A. Repeat US in 4 weeks.
B. Amniocentesis.
C. Foetal MRI.
D. Check maternal oestradiol levels.
E. It is a normal finding

A
  1. C. Foetal MRI

Ultrasound imaging is the screening modality of choice for initial evaluation of the foetal central nervous system. However, using MRI additional abnormalities were identified in 50% of the foetuses. Measurement of the hydrocephalus should be in the true axial plane at the atria of the lateral ventricle and glomus of the choroid plexus. The ventricle is measured from the inner margin of the medial ventricular wall to the inner margin of the lateral wall. Ventriculomegaly can be divided into three subgroups, borderline (10-12 mm), mild (>12-15 mm) and severe (>15 mm).

How well did you know this?
1
Not at all
2
3
4
5
Perfectly
10
Q

@# 16. Which of the following is not a recognised radiographic finding in a patient with haemochromatosis?
A. Chondrocalcinosis
B. Arthropathy with iron deposition in the synovium
C. General used increased hone density
D. Joint space narrowing
E. Osteophyte formation

A
  1. C. Generalised increased bone density

Haemochromatosis may either be primary or secondary. It is most commonly primary and congenital with an autosomal recessive (AR) inheritance. It is relatively common in Caucasian populations with an incidence of 1 in 300 to 1 in 400. Men are affected about 10 times more commonly than women, and at an earlier age.
It is often characterised radiographically by beak-like osteophytes projecting from the second and third metacarpal heads.
The other hallmark radiographic findings of haemochromatosis include the following: generalised osteoporosis (not increased bone density), arthropathy with iron deposition in the synovium (50%), joint space narrowing and enlargement of metacarpal heads. Chondrocalcinosis is also relatively common in this condition, most often affecting the knees and triangular fibrocartilage.

How well did you know this?
1
Not at all
2
3
4
5
Perfectly
11
Q

@# 17. A 9-year-old girl with a long history of cough, wheeze, sinusitis, headache and weight loss presented to the GP with an acute history of increasing breathlessness. Sweat test analysis shows 80 mmol/L of sodium chloride in forearm sweat (normal <40 mmol/L). All of the following are typical features on a chest X-ray, except
A. Hyperinflation
B. Bronchial dilatation
C. Cystic areas in the lung
D. Linear interstitial opacities
E. Dextrocardia

A
  1. E. Dextrocardia

Cystic fibrosis is an AR disorder leading to a defect in the CF transmembranc receptor (CFTR) protein resulting in defective ion transport in exocrine glands.

In CF, abnormal function of sweat glands result in higher concentrations of sodium chloride in the sweat; >40 mmol/L is suspicious and >60 mmol/L is diagnostic of CF.
Spirometry shows an obstructive pattern with reduced FVC and increased lung volumes. Chest X-ray shows hyperinflation, bronchial dilatation, bronchiectasis and its associated signs, cystic spaces, increases interstitial and linear/reticular opacities, and increased AP dimension on lateral chest X-ray. CF is not routinely associated with dextrocardia.
Dextrocardia is a component of Kartagener’s syndrome (immotile cilia syndrome), which is associated with bronchiectasis and sinusitis but not with an abnormal sweat test.

How well did you know this?
1
Not at all
2
3
4
5
Perfectly
12
Q

@# 19. A 62 year-old man with progressive cough and shortness of breath has bilateral patchy ground-glass change with areas of dependent and non-dependent septal thickening in
the basal lung zones on HRCT. Which of the following is incorrect about drug-induced lung disease?
A. Diffuse alveolar damage occurs with Gold.
B. Chronic nitrofurantoin toxicity results in high-density consolidation.
C High-density liver is seen in amiodarone toxicity.
D. There is no correlation between dose of methotrexate and toxicity.
E. NSIP is generally the most common change on HRCT.

A
  1. B. Chronic nitrofurantoin toxicity results in high density consolidation.

Nitrofurantoin is used to treat urinary tract infections. Acute pulmonary toxicity manifests
radiologically with diffuse bilateral, predominantly basal heterogeneous opacities. Non specific interstitial pneumonia (NSIP) is the most common histopathologic manifestation of chronic toxicity.
Methotrexate-induced pulmonary drug toxicity occurs in 5%-10% of patients. There is no correlation between the development of drug toxicity and the duration of therapy or total cumulative dose. NSIP is the most common manifestation; hypersensitivity pneumonitis and cryptogenic organising pneumonia (COP) are less common.
Diffuse alveolar damage and NSIP are the most common manifestations of gold-induced lung disease, with COP being less common.
NSIP is the most common manifestation of amiodarone-induced lung disease. Pleural effusion is recognised. COP is less common and occurs in association with NSIP. A distinctive feature of amiodarone toxicity is focal, homogeneous, peripheral, high-attenuation pulmonary opacities due to incorporation of amiodarone into Type II pneumocytes. The combination of high-attenuation abnormalities within the lung, liver or spleen is characteristic of amiodarone toxicity.

How well did you know this?
1
Not at all
2
3
4
5
Perfectly
13
Q

@# 28. All the following statements regarding HRCT changes in cytotoxic drug-induced lung disease are true, except
A. Bleomycin toxicity has a poor prognosis.
B. Paclitaxel is associated with pulmonary injury.
C. There is no correlation between carmustine dose and toxicity.
D. Diffuse alveolar damage (DAD) is the most common manifestation.
E. Bleomycin toxicity can be increased by concomitant oxygen therapy.

A
  1. C. There is no correlation between carmustine dose and toxicity.

Cyclophosphamide and busulfan are the most common drugs that cause lung injury. Diffuse
alveolar damage (DAD) is the most common manifestation of cyclophosphamide-induced lung disease, with NSIP and COP being less common. There is no relationship between development of lung injury and dose and duration of therapy.
Carmustine is one of the few drugs for which there is a clear relationship between cumulative dose and lung injury. Lung injury can occur at low doses if the patient has undergone thoracic radiation therapy. DAD is the most common manifestation, with NSIP being less common.
DAD is the most common manifestation of bleomycin-induced lung disease, with NSIP and COP being less common. The risk of developing lung injury is increased in the elderly, in patients on oxygen therapy, with a history of prior thoracic irradiation, or in whom therapy is restarted in 6 months of discontinuation. The prognosis is poor, with most patients dying of respiratory failure within 3 months.
Paclitaxel may commonly cause pulmonary toxicity.

How well did you know this?
1
Not at all
2
3
4
5
Perfectly
14
Q

@# 29. A 66-year-old person is sent for an urgent CT of the abdomen post-contrast. The CT
is unremarkable apart from inflammatory stranding in the omentum. No peritoneal nodules or ascites are detected on the CT and the report states possible omental infraction. All of the following are expected to be seen on the CT, except
A. Cake-like or whorled structure of mixed attenuation
B. Small amount of free peritoneal fluid
C. The fat-ring sign
D. Minimal reactive lymphadenopathy
E. Haziness of the fat anterior to the colon

A
  1. C. The fat ring sign

Omental infarction demonstrates a variety of imaging appearances on CT. Classically; it appears as a fatty, large (>5 cm) encapsulated mass, with soft-tissue stranding adjacent to the ascending colon. Early or mild infarction may manifest as mild haziness in the fat anterior to the colon.
Omental torsion is a rare cause of omental infarction and occurs when a portion of the omentum twists upon itself, leading to vascular compromise. In omental torsion, swirling of the vessels is often visible within the omentum. Although most cases of omental infarction are on the right side, left-sided infarction also may spontaneously occur.
Unusual locations of infarction are more commonly seen in the setting of surgical trauma or post-operative changes that result in altered omental vascular supply and subsequent infarction. Similar to epiploic appendagitis, the adjacent colon is usually spared, although rarely the colonic wall may be thickened, a result of direct extension of omental inflammation.

The fat-ring sign is seen in epiploic appendagitis, not omental infarction.

How well did you know this?
1
Not at all
2
3
4
5
Perfectly
15
Q

@# 31. Which one of the following statements regarding rheumatoid arthritis-associated thoracic manifestation is true?
A. Thoracic involvement occurs early in the disease.
B. Pleural effusions is usually bilateral.
C. COP is a recognised pattern on HRCT in rheumatoid lungs.
D. Fibrosis mostly affects the upper lobes.
E. Cavitation in nodules suggests some other disease.

A
  1. C. COP is a recognised pattern on HRCT in rheumatoid lungs.

Thoracic involvement develops in patients with disease progression. Pleural disease is the most common thoracic manifestation and pleural thickening is the most common finding, more than pleural effusion. Pleural effusions are usually unilateral and may be loculated. They usually occur late in the disease and are commonly associated with pericarditis and subcutaneous nodules. HRCT shows basal and peripheral predominant fibrosis. Rarely upper lobe fibrosis, mimicking tuberculosis (TB), can occur. There is increased prevalence of lung cancer in fibrotic rheumatoid lung disease. Nodules are multiple and well circumscribed, often forming thick-walled cavities. Obliterative bronchiolitis results in air trapping and mosaic pattern. COP is also seen on HRCT.

How well did you know this?
1
Not at all
2
3
4
5
Perfectly
16
Q

@# 33. A 45 year old man with proptosis of the right eye is referred for an MRI for further evaluation. MRI reveals a moderately large, well-defined intraconal retrobulbar mass, which is isointense to muscle on T1W images and markedly hyperintense to fat on T2W images. What is the most likely diagnosis?
A. Retinoblastoma
B. Rhabdomyoma
C. Haemangioma
D. Orbital pseudotumour
E. Metastatic lymphoma

A
  1. C. Haemangioma

Although not true neoplasms, cavernous malformations are the most common benign orbital mass in adults. Although these masses are commonly referred to as cavernous haemangiomas, many pathologists prefer the term cavernous malformation.
On CT images, cavernous malformations are typically well circumscribed, homogeneous and ovoid. The majority occur at the lateral aspect of the intraconal space. Conal and extraconal cavernous malformations are rare. Phleboliths are virtually never seen. Cavernous malformations tend to displace and surround adjacent structures, such as extraconal muscles and the optic nerve, rather than cause direct invasion. Osseous remodelling may be present, although bone erosion is rare. At MRI performed with T1-weighted sequences, cavernous malformations are isointense relative to muscle; the lesions appear uniformly hyperintense on T2-weighted images, with no flow voids. Internal septations may be identified on T2-weighted images. At multiphase CT, enhancement of cavernous malformations is poor on early arterial phase images, owing to the scant arterial supply. Delayed venous phase images demonstrate progressive filling of the mass from periphery to centre, with complete filling within 30 minutes. This enhancement pattern may permit differentiation of cavernous malformations from other vascular lesions with rich arterial supply, such as capillary’ haemangioma (a paediatric diagnosis), haemangiopericytoma and arteriovenous malformations.
Rhabdomyosarcoma is the most common soft-tissue malignancy of childhood and most common primary orbital malignancy. CT shows moderately well-defined to ill-defined margins, irregular shape and mild-moderate contrast enhancement. Adjacent bony destruction occurs in 40%. Globe distortion and extension to the paranasal sinuses may also be seen. Calcification is rare unless posttreatment. MR typically shows bright T2 signal, distinguishing rhabdomyosarcoma from other tumours such as chloroma (granulocytic sarcoma), lymphoma and metastatic neuroblastoma.

How well did you know this?
1
Not at all
2
3
4
5
Perfectly
17
Q

@# 34. A 32-year old woman is referred by her family doctor to the gynaecology clinic with history of congestive dysmenorrhoea, dyspareunia, fever and vaginal discharge. She had used a coil regularly in the past, which was later discontinued due to recurrent bouts of pelvic inflammation. Which one of the following statements concerning pelvic inflammatory disease is false?
A. Pyosalpinx can have a high Tl signal.
B. A tubular, fluid-filled structure with enhancing walls suggests pyosalpinx.
C. Delay in treatment can lead to tubo ovarian abscess.
D. Intense enhancement of inflamed areas on contrast-enhanced T1W MRI.
E. High signal m inflammatory stranding in peritubal fat on Tl fat-suppressed images.

A
  1. E. High signal in inflammatory stranding in peritubal fat on T1 fat-suppressed images

Pelvic inflammatory disease (PID) affects women of reproductive age and can lead to infertility, ectopic
pregnancy and chronic pelvic pain. In acute cases when dilated fallopian tubes are detected, it is extremely important to be able to differentiate tubal torsion from a pyosalpinx. A tubal torsion and a hematosalpinx may have a similar appearance to that of a fluid-filled tube on T2-weighted and STIR images; on T1-weighted images, however, a fluid filled tube has low signal intensity. Layering is common with haemorrhagic lesions, and tubal torsion may have a comma-shaped appearance. A pyosalpinx may have a similar appearance to that of a hydrosalpinx, but a hydrosalpinx usually has thinner walls. An abscess usually has low T1-weighted and high T2-weighted signal, but there can be a large variation in the signal intensity on T1-weighted and heterogeneity’ on T2-weighted images. Thick, irregular walls are typical of abscesses.
The Tl signal of pyosalpinx/TO abscess varies according to haemorrhagic/proteinaceous content On post-contrast T1-weighted images, pyosalpinx shows wall enhancement. Enhancement of the surrounding inflamed fat can also be evident. Inflammatory fat stranding is also seen on fat-suppressed T2-weighted sequences.

How well did you know this?
1
Not at all
2
3
4
5
Perfectly
18
Q

@# 35. A 40-year-old woman with short stature presents with early-onset hearing loss. Diagnostic workup reveals micromelic dwarfism, diffuse demineralisation and thinning of cortical bone, mild scoliosis and old fractures of the vertebral bodies and long bones. There was evidence of poor dentition. Which of the following is the most likely diagnosis?
A. Hypophosphatasia
B. Osteogenesis imperfecta
C. Paget disease
D. Osteoporosis
E. Achondroplasia

A
  1. B. Osteogenesis imperfecta

Osteogenesis imperfecta is an inherited disorder that results from mutations in either the COL1A1 or COL1A2 gene of Type I collagen. The disease is usually apparent at birth or in childhood, but more mild forms of the disease may not be apparent until adulthood. The disease is classified into Types I-IV, with Type I, the mildest form, being described in the question. The presenile hearing loss is caused by otosclerosis. The differential diagnosis can be resolved by the extraskeletal manifestations (blue sclerae and dentinogenesis imperfecta). The other types are Type II, lethal perinatal; Type III, severe progressive; and Type IV, moderately severe

How well did you know this?
1
Not at all
2
3
4
5
Perfectly
19
Q

@# 36. A macrosomic neonate (secondary to maternal diabetes) is noted to be in mild respiratory distress following delivery by caesarean section (CS). A chest X-ray demonstrates mild cardiomegaly mild hyperexpansion and small pleural effusions of the lungs. No focal lung abnormality is seen. What is the most likely diagnosis?
A. Respiratory distress syndrome
B. Meconium aspiration
C. Staphylococcal pneumonia
D. Group B streptococcal pneumonia
E. Transient tachypnea of the newborn

A
  1. E. Transient tachypnoea of the newborn

Although RDS is seen in association with maternal diabetes and caesarean section, hyperexpansion is not a feature of RDS. Transient tachypnoea of the newborn appears soon after birth (<4 hours) and has been identified as occurring with caesarean birth and infant sedation. Longer labour intervals, macrosomia of the foetus, and maternal asthma have also been associated. It may be accompanied by chest retractions, by expiratory grunting, or by cyanosis (which can be relieved with minimal oxygen). Recovery is usually complete within 3 days.
The lungs are usually affected diffusely and symmetrically, and the condition is commonly accompanied by small pleural effusions. The clinical course of transient tachypnoea is relatively benign when compared with the severity suggested by chest films. Radiographic resolution by the second or third day characterises this entity and differentiates it from other possible disorders; if radiographic resolution is not complete by the third day or if respiratory symptoms persist longer than 5 days, an alternative diagnosis should be sought.
Findings of transient tachypnoea of the newborn on chest radiographs may include mild, symmetrical lung overaeration, prominent perihilar interstitial markings and small pleural effusions. The radiographic appearance at times can mimic the diffuse, granular appearance of hyaline membrane disease but without pulmonary under aeration. Neonates with transient tachypnoea are usually delivered at term. Radiographic lung changes may also resemble the coarse, interstitial pattern seen with other causes of pulmonary oedema or the irregular pattern of lung opacification seen in meconium aspiration syndrome.

How well did you know this?
1
Not at all
2
3
4
5
Perfectly
20
Q

@# 37. A 6-year-old child is admitted to the emergency department presenting with diffuse abdominal pain, arthralgia and bilateral lower limb palpable purpura. Which of the following findings on an abdominal US examination is least expected?
A. Ileocaecal intussusception
B. Bowel wall thickening
C. Multifocal hepatic lesions
D. Bilaterally enlarged, echogenic renal cortices
E. Ascites

A
  1. C. Multifocal hepatic lesions

Henoch-Schönlein purpura is the most common vasculitis of childhood affecting small blood vessels. Clinical manifestations may include non thrombocytopenic purpura, arthritis, abdominal pain, gastrointestinal haemorrhage and glomerulonephritis. Imaging is useful to depict end-organ damage.
Ultrasound findings may include bowel wall oedema, submucosal and intramural haemorrhage, intussusception, hypoperistalsis, bowel dilatation, ascites, normal or enlarged echogenic kidneys, and also possible intramural haematomas within the urinary bladder and ureters. Non-specific findings on scrotal ultrasound may be identified such as hydroceles, scrotal wall thickening and inflammation of the spermatic cord and epididymis.

How well did you know this?
1
Not at all
2
3
4
5
Perfectly
21
Q

@# 40. A 33-year-old woman with progressive congestive dysmenorrhoea, deep dyspareunia and infertility is being evaluated for possible endometriosis. All of the following are typical MR findings of endometriosis, except:
A. Hypointense cysts on T1W images
B. Hyperintense cysts on T2W images
C. Multilocular cysts
D. Thick-walled cysts
E. Hypointense wall thickening on T1W and T2W images

A
  1. A. Hypointense cysts on T1W images

Findings of an adnexal mass with high T1-weighted signal and high T2 weighted signal (although slightly lower T2 signal than simple or functional cyst) is highly specific for an endometrioma. The main differential for high T1 -weighted cysts are haemorrhagic functional cysts and mature cystic teratoma. Cystic teratoma is differentiated by means of T1-weighted fat suppressed images. Endometriomas tend to have higher Tl and lower T2 signal than haemorrhagic cysts, due to higher protein content and viscosity (described as T2 shading). Multifocal lesions and bilateral lesions also favour endometriosis. Hemosiderin laden macrophages combined with the fibrous nature of the cyst wall give it a low signal-intensity appearance on both Tl - and T2 weighted images. Wall thickening, septae and nodularity are also recognised.

How well did you know this?
1
Not at all
2
3
4
5
Perfectly
22
Q

@# 41. A 6-year-old boy presents with increasing pain within his upper back, which came on insidiously over a few weeks. The child is otherwise well. A radiograph of his thoracic spine reveals collapse of the T9 vertebral body. The disc spaces are preserved; there is no kyphosis, and no involvement of the posterior elements. Which of the following is the most likely diagnosis?
A. Ewing’s sarcoma
B. Metastasis
C. Tuberculosis
D. Fracture
E. Eosinophilic granuloma

A
  1. E. Eosinophilic granuloma

The vast majority of ‘vertebra plana’ lesions in relatively healthy children are caused by an eosinophilic granuloma. The other available options are all possible, but less common, differential diagnoses. There is usually preservation of the disc space and no kyphosis. The posterior elements are rarely involved.

How well did you know this?
1
Not at all
2
3
4
5
Perfectly
23
Q

@# 44. A 62-year-old man, who has undergone solid organ transplant, is found to be profoundly neutropaenic and presents with progressive fever, cough and shortness of breath. He is acutely unwell. Chest X-ray shows bilateral patchy air space opacities and bronchoalveolar lavage showed Aspergillus organisms. HRCT is organised to confirm pulmonary invasive aspergillosis. All the following are typical imaging features, except:
A. Nodules with surrounding ground-glass change
B. Pulmonary sequestra
C. Peripheral wedge shaped consolidation
D. Air crescent sign
E. Central bronchiectasis

A
  1. E. Central bronchiectasis

Angioinvasive pulmonary aspergillosis occurs almost exclusively in immunocompromised patients with severe neutropenia. Characteristic CT findings include nodules surrounded by a halo of ground-glass attenuation (halo sign) or pleura-based, wedge-shaped areas of consolidation. In neutropenia patients, the halo sign is highly suggestive of angioinvasive aspergillosis. However, a similar appearance has been described in infection by Mucorales, Candida, herpes simplex virus (HSV), cytomegalovirus (CMV), Wegener’s granulomatosis, Kaposi’s sarcoma and haemorrhagic metastases. Separation of fragments of necrotic lung (pulmonary sequestra) from adjacent parenchyma results in air crescents similar to those seen in mycetomas. The air crescent sign is usually seen after initiation of treatment and with resolution of the neutropenia. Airway invasive aspergillosis shows multiple centrilobular nodules and the tree-in-bud pattern.

How well did you know this?
1
Not at all
2
3
4
5
Perfectly
24
Q

@# 45. A 2-year-old child presents with fever, erythema of the oral mucosa with chest and abdominal pain. Echocardiography reveals the presence of a coronary arterial aneurysms. An underlying vasculitis is suspected. Which of the following statements is least accurate in this clinical setting?
A. Aneurysms are typically seen in the proximal segments of the coronary arteries.
B. Aneurysms less than 5 mm in diameter are considered small.
C. Smaller aneurysms have a higher likelihood of thrombosis.
D. Multiple coronary artery aneurysms are more common than isolated aneurysms.
E. The most common site for a coronary aneurysm is in the left anterior descending artery.

A
  1. C. Smaller aneurysms have a higher likelihood of thrombosis

Kawasaki’s disease is a common paediatric vasculitis of medium-sized vessels, with coronary vasculitis being the hallmark manifestation. It is the leading cause of acquired heart disease in children in developed countries.
Coronary arterial aneurysms typically occur within the subacute phase of the disease and may be associated with sudden cardiac death. The aneurysms typically develop in the proximal segments of major coronary arteries and affect the left anterior descending artery followed by the proximal right coronary arteries in frequency of location. Smaller aneurysms, (<5 mm in diameter) are more likely to regress than larger aneurysms (>8 mm in diameter), which have a higher likelihood of thrombosis and infarction.

How well did you know this?
1
Not at all
2
3
4
5
Perfectly
25
Q

@# 46. A 61 -year old man with difficulty in swallowing was sent by his family doctor for a barium swallow. The examination showed a smooth filling defect in the mid lower oesophagus with minor hold up of contrast and proximal oesophageal dilatation.
CT performed for further evaluation did not show any extra oesophageal organ involvement or lymphadenopathy. The most common mesenchymal tumour of the oesophagus is
A. Lipoma
B. Gastrointestinal stromal tumour
C Haemangioma
D. Leiomyosarcoma
E. Leiomyoma

A
  1. E. Leiomyoma

Leiomyomas are neoplasms of mature smooth muscle cells and are the most common benign oesophageal neoplasm, although they are about 50 times less common than oesophageal carcinoma. They are also the most common mesenchymal tumours of the oesophagus, unlike in the remainder of the gastrointestinal tract, where GISTs predominate.
Oesophageal leiomyomas are nearly twice as common in men as m women and have been reported in patients between 4 and 81 years of age, although they rarely occur in the paediatric population. Most patients are asymptomatic, but dysphagia and pain may develop, depending on the size of the lesion and amount of encroachment on the oesophageal lumen, in contrast to patients with malignant oesophageal tumours.
Affected individuals usually have long-standing symptoms, with a duration of more than 2 years in most cases. Treatment options include endoscopic resection, surgical enucleation and observation. Oesophageal leiomyomas have a benign clinical course and typically do not recur after surgery.

How well did you know this?
1
Not at all
2
3
4
5
Perfectly
26
Q

@# 47. A 22 year old woman who had undergone a caesarean section presents with cyclic voiding symptoms hut no haematuria. Cystoscopy shows a filling defect and MRI is performed. Axial T2W MRI shows hypointense irregular focal wall thickening on the left posterolateral aspect of the bladder, without any fat stranding or associated lymph nodes, suggesting intrinsic bladder endometriosis. Which of the following statements is false, regarding deep pelvic endometriosis?
A. Subperitoneal invasion of endometriotic tissue must exceed 5 mm.
B. Endometriotic nodules can have high T2W’ signal.
C. Low signal nodular thickening of utcrosacral ligament on T2W images.
D. Ureteric endometriosis is mostly intrinsic.
E. Obliteration of pouch of Douglas is recognised in advanced disease.

A
  1. D. Ureteric endometriosis is mostly intrinsic.

Deep pelvic endometriosis is defined as subperitoneal invasion by endometriotic lesions that exceeds 5 mm in depth. Involvement of anatomic structures such as the uterosacral ligaments or the vaginal or rectal wall should be suspected when these structures have a hypointense thickened or nodular appearance on T2-weighted images. Intermingled high T2 signal would be secondary to ectopic endometrial glands. On T1-weighted or fat-suppressed T1 -weighted MR images, these foci may have either high or low signal intensity, depending on the presence or absence of bloody content. Some endometriomas may show restricted diffusion on DWI, probably due to intracystic blood clots. Enhancement may or may not occur post-contrast, depending on the proportions of inflammatory reaction, glandular tissue and fibrosis. Bladder endometriosis should be considered in anyone who presents with urinary tract symptoms after having undergone hysterectomy or other gynaecologic surgical procedure. Because vesical endometriosis seldom invades the mucosa, MR imaging may show abnormalities although cystoscopy is normal. As with bladder involvement, extrinsic endometriosis is the most common form of ureteral involvement. On MR imaging, ureteral endometriosis usually appears as irregular hypointense nodules on T2-weighted images. Deep retroperitoneal endometriotic lesions of the posterior compartment involve the rectovaginal pouch, retrocervical area, uterosacral ligaments, posterior vaginal fornix, rectovaginal septum and rectum. Obliteration of the pouch of Douglas occurs when retrocervical lesions extend to the anterior rectal wall.

How well did you know this?
1
Not at all
2
3
4
5
Perfectly
27
Q

@#e 48. A 7 year-old boy fell off a bike onto his outstretched left hand. X-ray showed a fracture of the left distal radius involving the epiphyseal plate that extends into the metaphysis. The epiphysis was split into two fragments. What is the Salter-Harris classification?
A. 1
B. 2
C. 3
D. 4
E. 5

A
  1. D. 4
    Physeal Salter-Harris fractures are divided into two categories on the basis of the involved physeal regions: (1) horizontal fractures without involvement of the germinal or proliferative zone of the physis and (2) longitudinal fractures that extend through all zones of the physis into the epiphysis.
    Horizontal fractures (Salter Harris Types I and II) result in bridge formation in 25% of cases, whereas longitudinal fractures (Salter-Harris Types III and IV) result in bridge formation in 75% of cases.
How well did you know this?
1
Not at all
2
3
4
5
Perfectly
28
Q

@# 49. An 81-year-old man with bilateral calcified and uncalcified pleural plaques, basal predominant fibrotic lungs and unilateral pleural effusion is sent for a CT scan with high suspicion of primary pleural mesothelioma. CT and pleural fluid aspirate confirmed mesothelioma. Which of these is not a feature of unresectability?
A. Extension into peritoneal cavity
B. Involvement of endothoracic fascia
C. Pericardial involvement
D. Multiple sites of extension into the chest wall
E. Direct extension into the ribs and spine

A
  1. B. Involvement of endothoracic fascia

Malignant mesothelioma occurs mainly in the pleura and peritoneum but can arise in the pericardium or tunica vaginalis testis. It is the most common primary neoplasm of the pleura and has a strong association with asbestos exposure, particularly crocidolite. Mesothelioma is not known to arise from plaques.
There is a new TNM staging system for diffuse malignant pleural mesothelioma that emphasises the criteria for determinate local tumour extension and regional lymph node status, to identify patients with potentially curable early disease (T1a and b); potentially resectable but not necessarily curable (T2 and T3); and unresectable local tumour spread (T4). Signs of unresectability include multifocal extension into chest wall with or without rib or spine destruction, extension through diaphragm into peritoneum, extension into contralateral pleura, extension into mediastinal organs, extension through pericardium and direct extension into heart. Localised tumour extension into the endothoracic fascia (T3) is potentially resectable.

29
Q

@# 50. A 70-year-old man with a history of weight loss and iron deficiency anaemia is referred for a CT colonography study. There is no malignancy on the scan but a 5 cm cystic structure is seen next to the distal ileum, in keeping with a duplication cyst The most common site of a duplication cyst of the gastrointestinal tract is the
A. Oesophagus
B. Stomach
C. Duodenum
D. Jejunum
E. Ileum

A
  1. E. Ileum

Enteric duplication cysts are an uncommon congenital abnormality. They can occur anywhere along the digestive tract on the mesenteric side. The small intestine is most commonly involved,
with the order from most to least common being the ileum, jejunum and duodenum. Most duplication cysts manifest during the first year of life, although some occasionally manifest in older patients. Children can present with a variety of symptoms including abdominal distention, vomiting, bleeding, a palpable abdominal mass and rarely urinary frequency and hesitancy. Complications include perforation, intussusception, bowel obstruction from adjacent pressure or mass effect, volvulus and associated malignancy. Malignant lesions arising from duplication cysts arc rare, particularly in children.

30
Q

@# 56. A 65 year old known diabetic woman is acutely unwell and presents with increasing epigastric tenderness. She is referred by the surgeons for an urgent contrast enhanced CT of the abdomen. The CT shows features consistent with emphysematous gastritis. All of the following are expected CT findings, except:
A. Air in the stomach wall
B. Pneumoperitoneum
C. Pneumobilia
D. Portal venous gas
E. Irregular gastric mucosal fold thickening

A
  1. C. Pneumobilia

Of all the hollow viscera, the stomach is the least commonly affected by gas forming infections. Of the 30 reported cases in the literature, caustic ingestion (37%) and alcohol abuse (22%) were found to be the most common causes. Other predisposing conditions include recent gastroduodenal surgery, trauma and gastric infarction. Caustic ingestion of acid is thought to promote coagulative necrosis of the gastric lumen, whereas ingestion of an alkaline substance leads to liquefactive necrosis; in either case, the end result is mucosal damage and super infection with gas-forming bacteria. There are no predilections with regard to age, sex or diabetic status. Clinical manifestation may be dramatic, ranging from acute sepsis to gastric haemorrhage and, rarely, vomiting of the necrotic stomach cast.
CT is considered the modality of choice for detection of intramural gas and evaluation for the presence of pneumoperitoneum or portal venous gas. CT may also demonstrate irregular mucosal fold thickening and may be used to monitor response to treatment or disease progression. An important differential diagnosis to consider is benign gastric emphysema. Gas collections form within the gastric wall without associated infection by gas-forming organisms. Gas may enter the wall from the lumen, peritoneal surface or oesophageal or duodenal connection and is usually associated with violent coughing, vomiting or severe obstructive pulmonary disease. Gastric fold inflammation and thickening are not present, and the patient is usually asymptomatic with spontaneous resolution expected.

31
Q

@# 57. A 31 -year-old woman was referred for a hysterosalpingogram for assessment of tubal anatomy and patency. The left fallopian tube was normal but no pelvic spillage was observed on the right, subject to proximal obstruction of the tube on the right. Which of the following conditions is not a cause of obstruction in the fallopian tube?
A. Granulomatous salpingitis
B. Intraluminal endometriosis
C. Congenital atresia
D. Peritubal adhesions
E. Tubal spasm

A
  1. D. Peritubal adhesions

The differential diagnosis of tubal occlusion typically includes tubal spasm, infection and prior surgery. Rare causes include granulomatous salpingitis due to tuberculosis, intraluminal endometriosis, parasitic infection and congenital atresia of the fallopian tubes. When tubal occlusion in the proximal or interstitial portion of the fallopian tube is seen at hysterosalpingography, a tubal spasm should be considered as the possible cause. Delayed radiography may be performed to help differentiate tubal spasm from true tubal occlusion. A spasmolytic agent such as glucagon also may be administered to relax the uterine muscle and relieve a tubal spasm.

Peritubal adhesions show’ peritubal pooling of contrast material rather than occlusion of fallopian tubes.

32
Q

@# 58. A 29-year-old man with history of discharge from the right ear underwent a CT of the temporal bones for further evaluation. Coronal CT images showed enlargement of the epitympanic recess and erosion of the walls and scutum with an associated soft-tissue mass. Only fragments of the ossicular chain could be identified. What is the diagnosis?
A. Cholesterol granuloma
B. Acquired attic cholesteatoma
C. Congenital cholesteatoma
D. Carcinoma of the middle car
E. Malignant otitis externa

A
  1. B. Acquired attic cholesteatoma

Cholesteatomas are composed of densely packed desquamated keratinizing squamous cells, arising from a peripheral shell of inward-facing epithelium. The role of CT is to assess the extent of disease and exclude complications.

Key findings on HRCT include soft-tissue opacification in the attic, aditus (non-dependent location) and/or mastoid air cells, erosion of the scutum, disruption of ossicular chain (long process of incus erosion is common), presence of disease in sinus tympani and erosions of the tegmen tympani, semicircular canal, facial nerve canal and inner ear. In cases of clinical suspicion of intracranial extension, perform contrast CT/MRI to rule out intracranial complications.

Congenital cholesteatoma is difficult to differentiate from acquired type, but clinical features help; it is commonly seen in children with intact tympanic membrane and absence of previous otologic disease.

33
Q

@# 59. An 8-year-old boy presents with a 5-week history of left hip pain and limp. Several previous pelvic X-rays were normal. A bone scan shows reduced uptake in the left femoral epiphysis. A line drawn from the lateral aspect of the femoral neck intersects the femoral head. What is the most likely diagnosis?
A. SUFE
B. Transient synovitis
C. Perthes disease
D. Developmental dysplasia of the hip
E. Sickle cell disease

A
  1. B. Transient synovitis

Acute transient synovitis is the most common non-traumatic cause of hip pain in young children. It tends to affect children between 2 and 9 years of age and boys are affected two to four times more often. Where history is typical no imaging may be required; ultrasound can be used to identify the effusion. Initial bone scan uptake can be reduced, mimicking SUFE, but can increase later on.

Juvenile arthritis affects children above 4-5 years of age. Radiographs may show erosions and loss of joint space. MRI and ultrasound are more sensitive for soft-tissue changes, allowing demonstration of synovitis, distinguishing pannus from simple effusion and identifying cartilage destruction and cortical erosions.

Septic arthritis is an emergency. The majority of patients are less than 2 years old and are usually unwell with pain on passive movement of the hip.

Avascular necrosis (AVN) of the femoral head is a condition induced by compromised blood supply, resulting in progressive destruction of bone. It is most commonly idiopathic (Perthes) but may be seen following trauma, infection, steroid treatment and in association with haematological diseases, such as sickle cell anaemia. Perthes disease usually affects children between 4 and 10 years of age and is more common in boys. Plain radiography is insensitive at early changes. The epiphysis may appear small, sclerotic or flattened with subchondral lucency or more marked fragmentation. Where symmetrical changes are seen, hypothyroidism or epiphyseal dysplasia should be considered.

Meyer’s dysplasia mimics Perthes disease unilaterally by appearance but is asymptomatic.

34
Q

@# 60. A 6-year-old boy with abdominal pain and jaundice was sent for a specialist review because the GP identified abnormal LFTs (raised bilirubin, raised AST and ALT). Viral serology for hepatitis B and C were negative. Twenty-four-hour urinary copper excretion was
>100 microgram (normal <40 microgram). An abnormal ring was noted in the eye on slit lamp examination. All of the following can be seen on US of the abdomen, except:
A. Mild hepatomegaly
B. Small echogenic liver
C. Abnormally large kidneys
D. Ascites
E. Abnormally large spleen

A
  1. C. Abnormally large kidneys

There are several causes of chronic liver failure in children: chronic hepatitis (hepatitis B and C), biliary atresia, drug induced (e.g., paracetamol), alpha-1-antitrypsin deficiency, Wilson’s disease, cystic fibrosis, IBD (inflammatory bowel disease), Budd-Chiari syndrome, TPN (total parenteral nutrition) induced and so on.
Abnormal sweat test (>60 mmol/L of sodium chloride) suggests CF, while reduced serum copper and caeruloplasmin and raised 24-hour urinary copper suggests Wilson’s disease. US shows hepatomegaly, echogenic liver, splenomegaly, ascites and varices.
Kayser Fleischer rings from copper deposition in the eye are pathognomonic but may require slit lamp examination to visualise. Adolescent patients can present with neurological symptoms.
Enlarged kidneys are not a feature of Wilson’s disease and may be seen in the acute phase of glomerulonephritis, ATN, renal vein thrombosis, amyloidosis, lymphoma, diabetes, glycogen storage disease, polycystic kidney disease and so on.

35
Q

@# 63. Which one of the following statements regarding penetrating aortic atherosclerotic ulcers is false? corrected
A. They can progress to pseodoaneurysm.
B. They are contrast-filled outpouchings surrounded by an intramural hematoma.
C. They occur early in atherosclerotic disease.
D. They can progress to aortic rupture.
E. Intramural haematoma signifies its aggressive nature.

A
  1. C. They occur early in atherosclerotic disease.

In a penetrating aortic ulcer (PAU), an atheromatous plaque ulcerates and burrows through the ultima into the aortic media. This leads to haemorrhage into the wall (intramural hematoma). The mural haematoma may break through into the adventitia to form a pseudoaneurysm, or it may rupture.

Ulceration of an aortic atheroma occurs in patients with advanced atherosclerosis.

On imaging, a penetrating aortic ulcer can be distinguished from an atheromatous plaque by presence of a focal, contrast-filled outpouching surrounded by an intramural hematoma, which confirms the aggressive behaviour of the lesion.

The atheromatous plaque with ulceration but without penetration through the intima shows irregular margins, but no contrast material extends beyond the level of intima, which is frequently calcified, and no intramural hematoma is present.

PAU can present with chest or back pain.

36
Q

@# 65. A 26-year-old man with facial pain shows a large, well-demarcated expansile cystic lesion in the petrous apex with erosion of the internal auditory meatus. No associated erosion of the scutum is identified, and the tympanic membrane is intact. MRI reveals a corresponding high signal lesion on both T1W and T2W sequences. What is the diagnosis?
A. Cholesterol granuloma
B. Acquired cholesteatoma
C. Congenital cholesteatoma
D. Carcinoma of petrous temporal bone
E. Arachnoid cyst

A
  1. A. Cholesterol granuloma

Cholesterol granulomas of the temporal bone can occur in the mastoid segment, the middle ear and the petrous apex. They are the most common primary’ petrous apex lesions.

Temporal bone CT reveals an expansile, sharply defined and often rounded mass of the petrous apex with cortical thinning and trabecular breakdown. The general appearance is that of a slowly progressive benign process. There is central soft-tissue density without an internal matrix, a calcification or residual septations. If the lesion is sufficiently enlarged, frank bony dehiscence is observed.

On MRI, cholesterol granulomas are typically hyperintense on both T1 and T2-weighted sequences because of the accumulation of blood breakdown products and proteinaceous debris. Small lesions may be relatively homogeneous, whereas large lesions show more heterogeneity.

Often cholesterol granulomas have a distinct hypointense peripheral rim on T2-weighted images due to hemosiderin deposition.

After contrast administration, there may be subtle peripheral enhancement secondary’ to inflammatory response but no central enhancement that would indicate solid tissue.

37
Q

@# 66. A 6-year-old girl complains of pain in her left shoulder. X-ray shows a lucent lesion in the proximal humeral epiphysis. The lesion has sclerotic borders with specks of calcification within the lesion. What is the most likely diagnosis?
A. Simple bone cyst
B. Aneurysmal bone cyst (ABC)
C. Non-ossifying fibroma
D. Fibrous cortical defect
E. Chondroblastoma

A
  1. E. Chondroblastoma

Chondroblastomas are rare, benign, cartilaginous tumours that affect the epiphysis of children. On MR images, chondroblastomas are seen as epiphyseal lesions with high T2 signal intensity surrounded by a halo of oedema in the adjacent marrow and soft tissues. A characteristic thin (<1 mm) low-signal-intensity ring that corresponds to peripheral sclerosis is seen in more than 90%. Fluid fluid levels similar to Aneurysmal bone cyst (ABCs) are seen in 20%-30% of cases. Differential considerations include epiphyseal osteomyelitis and osteoid osteoma.
Neuroblastoma metastasis and Langerhans cell hystiocytosis can also affect the epiphytes.

Simple bone cyst (SBC), fibrous cortical defects and ABC affect the metaphysis.

38
Q

@# 67. Which one of the following extra-intestinal manifestations of inflammatory bowel disease is more likely to occur in patients with Crohn’s disease than in those with ulcerative colitis?
A. Sclerosing cholangitis
B. Pyoderma gangrenosum
C. Gallstones
D. Erythema nodosum
E. Pulmonary embolism

A
  1. D. Erythema nodosum

Pyoderma gangrenosum (PG) has been reported in 1%—10% of ulcerative colitis (UC) patients and 0.5%-20% of Crohn’s disease (CD) patients. There is no significant difference between genders. There have been conflicting data regarding the distribution of PG among CD and UC; however there is no statistically significant difference between the two groups.

Erythema nodosum is more common in CD than in UC. The occurrence of lesions parallels intestinal disease activity, and lesions frequently resolve when bowel disease subsides; thus, treatment is usually aimed at the underlying bowel disease. At times, erythema nodosum can precede bowel exacerbations and can require treatment with oral steroids. Based on these findings, patients with idiopathic EN should be evaluated for IBD.

39
Q

@# 69. A 66-year old man with acute onset of chest pain radiating to the back is sent for an urgent post-contrast CT of the chest to exclude aortic dissection. There is no evidence of a dissection or a penetrating aortic ulcer, but the scan is reported as showing an acute intramural haematoma in the descending thoracic aorta. All of the following are characteristic CT findings, except:
A. It is subintimal in location.
B. It appears as a crescent of high density.
C. MRI can help differentiate it from slow flow in false lumen of dissection.
D. The aortic wall appears thinned out.
E. They can progress to aortic rupture.

A
  1. D. The aortic wall appears thinned out.

Aortic intramural hematoma may occur as a primary event in hypertensive patients from spontaneous bleeding from the vasa vasorum into the media or may be caused by a penetrating atherosclerotic ulcer.

Intramural hematoma can also develop in blunt chest trauma with aortic wall injury. The hematoma propagates along the media layer of the aorta. Consequently, intramural hematoma weakens the aorta and may progress either to outward rupture of the aortic wall or to inward disruption of the intima, the latter leading to communicating aortic dissection.

Intramural hematoma can be distinguished from mural thrombus by identification of the intima: mural thrombus lies on top of the intima, which is frequently calcified, whereas intramural hematoma is subintimal.

On unenhanced CT, intramural hematoma is hyperdense. MR imaging aids in the distinction of slow flow in the false lumen of a dissection from no flow in an intramural hematoma. Dynamic post-contrast MR imaging is more sensitive for excluding slow flow in the thickened aortic wall, which would indicate aortic dissection rather than intramural hematoma.

40
Q

@# 72. On a routine neonatal clinical examination, the paediatric registrar notes a simple sacral dimple at the natal cleft. It is blind-ended without any associated tuft of hair at the site of the dimple. What is the most reasonable subsequent management plan?
A. No further management is required.
B. Ultrasonography of the spine.
C. Ultrasonography of the spine and cranial contents.
D. MRI of the spine.
E. MRI of the brain and spine

A
  1. A. No further management is required.

Typical indications for spinal US in newborns and infants are skin covered masses and midline cutaneous malformations of the back (e.g., dimple, haemangiomatous or hairy lesion), which are suggestive of associated dysraphic anomalies of the spinal cord. Spinal dysraphism is often associated with tethering of the spinal cord. The US appearance of tethering is a low-lying or blunt-ended conus medullaris due to abnormal fixation of the spinal cord. Moreover, movement of the spinal cord and cauda equina can be evaluated with real-time US with M mode scanning. Typically, the tethered cord is positioned eccentrically and demonstrates reduced or absent movement. Dorsal dermal sinus manifests as a small dimple or pinpoint ostium, which is often associated with an area of hyperpigmented, angiomatous skin or hypertrichosis and occurs in a midline location or rarely in a paramedian location. Soft tissue asymmetry and bone anomalies are common findings. Typical complications are infections such as recurrent meningitis, epidural or subdural abscess, and intramedullary spinal cord abscess. In particular, dorsal dermal sinus occurring in a paramedian location is often associated with an intraspinal dermoid or epidermoid cyst, which causes compression of neural structures with neurologic symptoms. For these reasons, dorsal dermal sinus has to be differentiated from simple sacral dimple or pilonidal sinus. The latter two anomalies do not extend to neural structures.

41
Q

@# 73. A 4-year-old child falls onto an outstretched arm while on the playground. A radiograph of the elbow demonstrates the presence of a posterior fat pad adjacent to the distal humerus. There is no cortical defect or obvious fracture on the radiograph. What is the most likely underlying pathology?
A. Osteomyelitis
B. Supracondylar fracture
C. Head of radius fracture
D. Clinoid fracture of the ulna
E. Septic elbow joint effusion

A
  1. B. Supracondylar fracture

The value of the fat pad sign is greatest as a predictor of an intra-articular disease process at the elbow in the absence of any radiographically visible bone abnormality. Fat pad displacement is independent of fracture displacement and comminution. This applies in particular to elbow examination in children, who often have very slight structural changes at presentation. Supracondylar fractures account for 60% of all elbow fractures in children, followed by fracture of the lateral epicondyle (15%) and separation of the medial epicondylar ossification centre (10%). In adults, fracture of the radial head or neck accounts for just under 50% of all fractures at the elbow, followed by fracture of the olecranon (20%) and dislocations and fracture dislocations.

42
Q

@# 74. All of the following structures are present in the centre of the secondary pulmonary lobule, except:
A. Respiratory bronchiole
B. Pulmonary artery
C. Pulmonary vein
D. Lymphatics
E. Bronchovascular interstitium

A
  1. C. Pulmonary vein
    The secondary pulmonary lobule, as defined by Miller, refers to the smallest unit of lung structure marginated by connective tissue septa. Secondary pulmonary lobules are irregularly polyhedral in shape and vary in size, measuring from 1 to 2.5 cm in diameter.
    Airways, pulmonary arteries and veins, lymphatics and the various components of the pulmonary interstitium are all represented at the level of the secondary lobule. Each secondary lobule is supplied by a small bronchiole, pulmonary artery and lymphatic branches centrally and is marginated by connective tissue, the interlobular septa, that contain pulmonary veins and lymphatics.
43
Q

@# 77. Which technique reduces artefact from hip prosthesis during MRI?
A. Use of a magnet with a higher field strength
B. Use of FSE imaging rather than GE imaging
C. Alignment of prosthesis perpendicular to the magnetic field
D. Using a narrower bandwidth
E. Use of the magic angle

A
  1. B. Use FSE imaging rather than GE imaging
    Several strategies help in reduction of susceptibility artefact from hip prosthesis. Some of these are as follows:
    * Reduced magnetic field strength
    * Increasing bandwidth during slice selection and readout
    * Increasing matrix size: 512 pixels
    * Maintain good SNR (signal to noise ratio) by increasing number of excitations NEX (number of excitation)
    * Use spin echo (FSE) instead of gradient echo (GRE) where possible
    * STIR for fat suppression (spectral frequency selective fat suppression performs better in a homogeneous field)
    * Use of shorter echo spacing
    * Use smaller water-fat shift
    * Use thinner slices
    * Align prosthesis parallel to the magnetic field
    * Use view-angle tilting (VAT)
44
Q

@# 78. A 19-year-old man shows a large mass in the nasopharynx on CT sinuses. Post-contrast images confirmed an intensely vascular mass extending into the pterygopalatine fossa and infratemporal fossa with forward displacement of the posterior wall of the maxillary sinus. Coronal images showed erosion of the base of the pterygoid plate. What is the diagnosis?
A. Inverted papilloma
B. Nasopharyngeal carcinoma
C. Juvenile angiofibroma
D. Lymphoma
E. Midline granuloma

A
  1. C. Juvenile angiofibroma

Juvenile angiofibroma presents characteristic imaging signs. The diagnosis by CT is based upon the site of origin of the lesion in the pterygopalatine fossa. There are two constant features:
(1) a mass in the posterior nasal cavity and pterygopalatine fossa; (2) erosion of bone behind the sphenopalatine foramen with extension to the upper medial pterygoid plate.
The characteristic features on MRI are due to the high vascularity of the tumour, causing signal voids and strong post contrast enhancement.

45
Q

@# 81. A 66 year old man presented to the A&E department with progressive chest tightness and shortness of breath. ECG was normal and inflammatory markers were normal. Chest X-ray was abnormal and an HRCT was requested. HRCT showed a diffuse interstitial pattern of small nodular opacities. All of the following may be associated with this finding, except:
A. Sarcoidosis
B. Talcosis
C. Scleroderma
D. Hypersensitivity pneumonitis
E. Small nodular metastasis

A
  1. C. Scleroderma

There are three possible HRCT distributions of nodules: perilymphatic, random and centrilobular.
The causes of perilymphatic nodules include sarcoidosis, lymphangitis carcinomatosa, silicosis, amyloidosis (rare) and lymphoid interstitial pneumonia (rare). The causes of random nodules include miliary TB, miliary spread of fungal infection, metastasis and sarcoidosis (rare). The causes of centrilobular nodules include endobronchial infection, endobronchial tumour, aspiration, respiratory bronchiolitis interstitial lung disease (RB-ILD), hypersensitivity pneumonitis (HP), histiocytosis, vascular causes (oedema and haemorrhage) and talcosis.

46
Q

@# 82. A 37-year old woman with known history of inflammatory bowel disease presented to the gastroenterologist with worsening abdominal and systemic symptoms. All of the following are extra-intestinal musculoskeletal manifestations of inflammatory bowel disease, except:
A. Neuropathic osteoarthropathy
B. Enteropathic spondylitis
C. Hypertrophic osteoarthropathy
D. Sacroiliitis
E. Monoarticular peripheral arthritis

A
  1. A. Neuropathic osteoarthropathy
    Extra intestinal manifestations of inflammatory bowel disease:
    Musculoskeletal System Arthritis
    Colitic type, ankylosing spondylitis, isolated joint involvement, hypertrophic osteoarthropathy: clubbing, periostitis
    Miscellaneous Manifestations Osteoporosis, aseptic necrosis, polymyositis Dermatologic and Oral Systems Reactive Lesions
    Erythema nodosum, pyoderma gangrenosum, aphthous ulcers, necrotising vasculitis Specific Lesions
    Fissures, fistulas, oral Crohn’s disease, drug rashes Nutritional Deficiencies
    Acrodermatitis enteropathica, purpura, glossitis, hair loss, brittle nails
    Associated Diseases
    Vitiligo, psoriasis, amyloidosis
    Hepatopancreatobiliary System
    Primary sclerosing cholangitis, bile duct carcinoma Associated Inflammation
    Autoimmune chronic active hepatitis, pericholangitis, portal fibrosis, cirrhosis, granulomatous disease Metabolic Manifestations
    Fatty liver, gallstones associated with ileal Crohn’s disease Ocular System
    Uveitis/iritis, episcleritis, scleromalacia, corneal ulcers, retinal vascular disease Metabolic System
    Growth retardation in children and adolescents, delayed sexual maturation
    Renal System
    Calcium oxalate stones
47
Q

@# 83. On your routine MR reporting session, a lumbar spine shows normal alignment and multilevel low signal on both T1 and T2 in the intervertebral discs centrally, with disc space narrowing. What is the most likely diagnosis?
A. CPPD
B. Ochronosis
C. Amyloidosis
D. Renal osteodystrophy
E. Gout

A
  1. B. Ochronosis

Causes of intervertebral disc calcification include the following:
* Degenerative disc disease is a relative common cause for disc calcification.
* Alkaptonuria, or ochronosis, results in dense central calcification affecting the nucleus pulposus and is associated with generalised osteopaenia. Changes often start at the lumbar spine.
* Ankylosing spondylitis is a recognised cause; associated findings helping in narrowing the diagnosis.
* Calcium pyrophosphate dehydrate deposition disease (CPPD), haemochromatosis and hypervitaminosis D can result in calcification of the annulus fibrosus.
* Transient intervertebral disc calcification is seen in children, typically in the cervical spine and spontaneously regresses.
* Other recognised causes of disc calcification include juvenile chronic arthritis, amyloidosis, poliomyelitis, acromegaly, hyperparathyroidism, trauma and post-operative discs.

48
Q

@# 86. A 67-year-old man with worsening abdominal pain and LFTs shows a peripheral mass in the right lobe of the liver. Contrast-enhanced dynamic MRI done for further characterisation confirmed the lesion seen on US, with evidence of liver capsular retraction consistent with
desmoplastic reaction commonly associated with peripheral eholangiocarcinoma. All of the following are risk factors, except:
A. Radium exposure
B. Chronic hepatitis
C. Primary sclerosing cholangitis
D. Thorotrast exposure
E. Clonorchis sinensis infection

A
  1. B. Chronic hepatitis

There are a number of recognised risk factors for cholangiocarcinoma that all share the common feature of chronic biliary inflammation. Among these risk factors, infection with liver flukes (e.g., Opisthorchis viverrini and Clonorchis sinensis) and hepatolithiasis arc common causes of cholangiocarcinoma in endemic areas. Dietary or endogenous nitrosamine compounds associated with parasitic infections also play an important role as cofactors in carcinogenesis, probably due to the carcinogenic effect of nitrosamine compounds on the proliferation of epithelial cells of the bile duct Cholangiocarcinoma arising from a cirrhotic liver may be surrounded by a fibrotic pseudocapsule, which is an unusual finding in cholangiocarcinoma arising from a non-cirrhotic liver. In such cases, capsular retraction is noted along the tumour surface. This capsular retraction may be seen in some hepatocellular carcinomas (HCCs) with cirrhotic stroma but is more suggestive of cholangiocarcinoma. Cholangiocarcinoma can develop in a congenital choledochal cyst, with a lifetime risk of 10%-15%. In addition, a European study showed that a history of alcohol-related liver disease, cirrhosis, various bile-duct diseases, chronic inflammatory bowel disease or diabetes may increase the risk of development of cholangiocarcinoma.

49
Q

@# 87. A 59-year-old woman with an apparent lump in the lower abdomen, weight loss and new onset tremor was sent for an MRI of the pelvis to investigate. Sagittal T2W MRI showed a multiloculated, heterogeneous left ovarian lesion with very low signal intensity. Corresponding axial in-phase and out-of-phase images revealed a hypointense mass with chemical shift artefact in its ventral aspect. Axial post contrast T1W MRI showed significant enhancement of the ovarian lesion. What is the diagnosis?
A. Brenner tumour
B. Struma ovarii
C. Ovarian thecofibroma
D. Mucinous cystadenoma
E. Endometrioma

A
  1. B. Struma ovarii

Struma ovarii is a rare ovarian lesion that accounts for 2% of ovarian teratomas.

Struma ovarii is a highly specialised form of ovarian teratoma and is composed entirely or predominantly of thyroid tissue. About 5% of patients develop clinical evidence of hyperthyroidism. At US, struma ovarii has a non-specific solid, cystic appearance. MRI demonstrates a loculated cystic mass with variable signal characteristics. Cystic spaces may show marked T2 hypointensity and intermediate Tl signal intensity due to the thick, gelatinous colloid of the struma.

Some locules may contain microscopic fat, as indicated by signal drop- off and chemical shift artefact on opposed-phase T1-weighted MRI. Struma ovarii typically demonstrates strong enhancement of the solid components on post-contrast T1-weighted MRI.

Struma ovarii are benign in 95% of cases and usually occur in premenopausal women; therefore, preoperative diagnosis is essential to avoid unnecessary radical surgery.

50
Q

@# 88. Posteromedial corner (PMC) of the knee involves the following structures:
A. Semimembranosus tendon and posterior oblique ligament
B. Popliteus muscle and arcuate ligament
C. Iliotibial band and biceps femoris
D. Gastrocnemius and PCL
E. Medial retinaculum and medial collateral ligament

A
  1. A. Semimembranosus tendon and posterior oblique ligament

The PMC contains the structures lying between the posterior margin of the longitudinal fibres of the superficial medial collateral ligament and the medial border of the posterior cruciate ligament. The PMC has five major components: the semimembranosus tendon and its expansions, the oblique popliteal ligament (OPL), the posterior oblique ligament (POL), the posteromedial joint capsule (or simply the posteromedial capsule) and the posterior horn of the medial meniscus.

51
Q

@# 90. While reporting a plain radiograph of the knee in a 5-year-old child, you notice premature closure of the distal femoral growth plate. Which of the following features in the clinical history would not explain this?
A. Localised radiotherapy to the leg
B. Hypervitaminosis A
C. Osteomyelitis
D. Previous trauma to the knee
E. Hyperparathyroidism

A
  1. E. Hyperparathyroidism
    Physeal fracture is the most common cause of bone bridging across the growth plate, but growth arrest may also be due to other insults: infection, therapeutic irradiation, metabolic or haematologic abnormality, tumour, bum, frostbite, electrical injury’, sensory neuropathy, microvascular ischaemia or insertion of metal. Premature fusion of the growth plate has also been reported in patients with hypervitaminosis A.
52
Q

@# 92. A 67-year-old man with a progressive restrictive pattern of pulmonary function test and increasing chest tightness was sent for an HRCT for characterisation. HRCT showed nodular septal thickening with further bronchovascular nodules and thickening. Which one of the following disorders is most likely to be the cause?
A. Pneumoconiosis
B. Lymphangitis
C. Lymphocytic interstitial pneumonia
D. Hypersensitivity pneumonitis
E. Churg-Strauss syndrome

A
  1. B. Lymphangitis
    The most common malignancy associated with lymphangitis carcinomatosa is bronchogenic carcinoma, most commonly adenocarcinoma, followed by breast, GI malignancies (stomach and colon) and prostate cancer.
    Chest CT is more specific and sensitive than CXR for the diagnosis of lymphatic metastasis. On HRCT, there is variable, smooth, irregular or nodular thickening of the interlobular septae and bronchovascular bundles, which often have a beaded appearance. Another characteristic feature is either smooth or nodular thickening of the peribronchovascular interstitium. Similar changes can be seen along the fissures. In many patients, the abnormality is unilateral or patchy. Pleural effusion and hilar adenopathy are associated.
53
Q

@# 93. A 48-year-old man with a history of alcohol excess is admitted with a variceal bleed. Regarding upper gastrointestinal varices, which of the following statements is correct?
A. They are the result of hepatopetal blood flow from the left gastric vein and splenic vein to the superior mesenteric vein.
B. Oesophageal varices tend to bleed more severely than gastric varices.
C. Splenic portography is the first-line investigation for assessment.
D. Barium studies can detect gastric varices in approximately 75% of cases evidenced by lobulated folds and polypoidal fundal masses.
E. Gastric varices bleed more frequently than oesophageal varices.

A
  1. D. Barium studies can detect gastric varices in approximately 75% of cases evidenced
    by lobulated folds and polypoidal fundal masses

Gastrointestinal varices occur as a result of hepatofugal flow from the left gastric vein and splenic vein to the superior mesenteric vein. This usually occurs secondary to liver cirrhosis but isolated splenic vein occlusion (with pancreatic disease) can also be the cause.

Gastric varices bleed less frequently but more severely than oesophageal varices.

54
Q

@# 95. Which of the following anatomical variants and its associated clinical symptom is incorrect?
A. Os acromiale - impingement
B. Conjoint spinal nerve roots - muscle weakness
C. Positive ulnar variance - TFC tear
D. Discoid meniscus - locking
E. Os naviculare - pain behind the heel

A
  1. E. Os naviculare - pain behind the heel
    The os acromiale has been implicated as a risk factor for the development of impingement syndrome. Hypertrophic osteophytes may arise at the synchondrosis of an os acromiale, and the os acromiale is thought to increase the incidence of osteoarthritis at the AC joint.
    Medial side foot pain (os naviculare syndrome) is the most common presenting feature of accessory navicular bone; the pain is aggravated by walking, running and weight-bearing activities.
    Positive ulnar variance is associated with ulnar impaction syndrome or ulnocarpal abutment with TFC degeneration and ulnar-sided wrist pain.
    Discoid meniscus is an uncommon anatomical variant, more commonly affecting the lateral meniscus. Although frequently asymptomatic, it is prone to cystic degeneration with subsequent tears. Clinical presentation may be with pain, locking or clicking.
    All lands of neurological deficits and clinical symptoms may occur with conjoined nerve roots. Besides the different phenotypes of low back or sciatic pain, the most common complaints are numbness and muscular weakness.
55
Q

@# 96. A child is referred with a presumed diagnosis of a mucopolysaccharidosis. A skeletal survey demonstrates multiple features in keeping with dysostosis multiplex. Which one of the following radiographic features is not part of this syndrome?
A. J-shaped sella
B. Tilting of the radius and ulna away from each other
C. Arrowhead terminal phalanx
D. Calvarial thickening
E. Proximal pointed metacarpals

A
  1. C. Arrowhead terminal phalanx

Mucopolysaccharidoses (MPS) represent a heterogeneous group of inheritable lysosomal storage diseases in which the accumulation of undegraded glycosaminoglycans (GAGs) leads to progressive damage of affected tissues. The typical symptoms include organomegaly, dysostosis multiplex, mental retardation and developmental delay.

Dysostosis multiplex is represented by several bone malformations found in the skull, hands, legs, arms and column. Some of the other common skeletal manifestations include macrocephaly with dolichocephaly, facial anomalies, obtuse angle of mandible with prognathism, paddle- or oar-shaped ribs, atlanto-axial instability, malformed vertebral bodies, hip dysplasia, coxa valga, proximal humeral notching, inferior tapering of ileum, rounded iliac wings, bullet shaped phalanges and hypoplastic and irregular carpal and tarsal bones. The abnormal storage of GAGs leads to liver and spleen enlargement; it also damages cartilage layers and synovial recesses in the joints.

56
Q

@# 98. A 65-year-old man is admitted with intractable retching and sudden onset epigastric pain.
A nasogastric tube could not be passed into the stomach. Regarding gastric volvulus, which one of the following statements is correct?
A. Aetiology is related to unusually short gastrohepatic and gastrocolic mesenteries.
B. Sliding/para-oesophageal hernia does not predispose to gastric volvulus.
C. Organo-axial volvulus is more common than mesentero-axial volvulus and creates a ‘mirror-image’ stomach.
D. Mesentero-axial volvulus rotates around a line from cardia to pylorus.
E. Organo-axial volvulus is associated with ‘upside down stomach’ appearance

A
  1. C. Organo-axial volvulus is more common than mesentero-axial volvulus and creates a
    ‘mirror-image’ stomach

The aetiology of gastric volvulus is related to unusually long gastrohepatic and gastrocolic mesenteries.

Organo axial volvulus rotates around a line from the cardia to the pylorus, whereas the axis of rotation in mesentero-axial volvulus runs from the lesser to the greater curve of stomach.

Mesentero-axial volvulus is associated with ‘upside-down’ stomach appearance, whereas organo-axial volvulus is associated with a ‘mirror-image’ stomach.

57
Q

@# 99. A 31-year-old woman with a complex cystic adnexal mass on the left identified on pelvic ultrasound was referred for a pelvic MRI with a view towards further characterisation of the mass lesion. Which of the following statements regarding ovarian tumour is incorrect?
A. Granulosa cell tumour is associated with endometrial hyperplasia.
B. The presence of fat is specific for teratoma.
C. Immature teratoma is mainly cystic.
D. Dysgerminoma is associated with raised HCG level.
E. Brenner tumour is associated with calcification.

A
  1. C. Immature teratoma is mainly cystic.

Ovarian tumours associated with endometrial hyperplasia or carcinoma includes endometrioid carcinoma, granulosa cell tumour and, occasionally, thecoma or fibrothecoma.

Although rare, endometrioid carcinoma is the most common malignant neoplasm that arises from endometriosis.

The presence of fat opacity or fat signal intensity in an ovarian lesion is highly specific for a teratoma.

Mature cystic teratomas are predominantly cystic with dense calcifications, whereas immature teratomas are predominantly solid with small foci of lipid material and scattered calcifications.

Ovarian tumours that are frequently associated with calcifications include serous epithelial tumour, fibrothecoma, mature or immature teratoma and Brenner tumour.

Malignant germ cell tumours include dysgerminoma (raised HCG level) and endodermal sinus tumours (α-fetoprotein level) and are found in younger patients

58
Q

@# 100. A 48-year-old woman with mid and lower back pain and shortness of breath presented to the the A&E department. Plain radiographs showed diffusely osteopaenic bones and an old superior end plate depression fracture of L1. CTPA showed acute pulmonary emboli. Plain X-ray of her hand done a year ago at a different hospital showed a metacarpal index of 9.8. What is your diagnosis?
A. Marfan’s syndrome
B. Homocystinuria
C. Systemic lupus erythematosus
D. Acromegaly
E. Soto’s syndrome

A
  1. B. Homocystinuria

Homocystinuria is an AR disorder secondary to deficiency of cystathionine synthase. Arachnodactyly (metacarpal index >8.4 or >9.4; depending reference standard used) is seen in one in three patients (cf. 100% in Marfan syndrome).

Lens dislocation is downwards and inwards
(cf. upwards and outwards in Marfan syndrome).

Homocystinuria is also associated with osteoporosis, bowing/factures, pectus deformities and biconcave vertebra.

There is increased propensity of thromboembolic phenomena due to increased stickiness of platelets. Death is often from occlusive vascular disease.

Sotos syndrome is an autosomal dominant syndrome considered a form of cerebral gigantism.

59
Q

@# 101. A 22-year-old man with a rapidly enlarging painful right maxilla showed an opacified right maxillary antrum on plain radiograph with destruction of the lateral wall. Axial CT showed extensive new bone formation on both sides of the anterolateral wall of the maxillary antrum with sun ray spiculations anteriorly. What is the diagnosis?
A. Ewing’s sarcoma
B. Synovial sarcoma
C. Antral carcinoma
D. Myeloma
E. Osteogenic sarcoma

A
  1. E. Osteogenic sarcoma

Fewer than 10% of osteosarcomas arise in the craniofacial bones, with most such tumours developing in the mandible and maxilla. The most common sites of involvement are the body of the mandible and the alveolar ridge or the antral area of the maxilla. It may be secondary to radiation, fibrous dysplasia, Paget disease, trauma, osteomyelitis, ossifying fibroma and giant cell tumour.
On CT, the tumour displays a spectrum of bone changes from well-demarcated borders, notably the low-grade osteosarcoma (uncommon), to lytic bone destruction with indefinite margin and variable cortical bone erosion, to the osteoblastic form, where the bone is sclerotic. The majority of osteosarcomas have matrix mineralisation, calcifications of the osteoid or osteoid like substance within the tumour and some tumours show a sunburst effect caused by radiating mineralised tumour spiculae.
Ewing’s sarcoma can also occur in this area, although the expected age would be younger.
On CT, it often shows the characteristic onion-skin appearance of periosteal reaction and less often a sunburst type of periosteal reaction.

60
Q

@# 102. A chest radiograph of a 3-year-old child demonstrates marked right lower zone consolidation with a large pneumatocoele. A diagnosis of necrotising pneumonia is made. What is the most likely causative organism?
A. Staphylococcus aureus
B. Streptococcus pyogenes
C. Bordatella pertussis
D. Mycobacterium tuberculosis
E. Aspergillus

A
  1. A. Staphylococcus aureus

Pneumatocoeles are thin-walled, air filled intraparenchymal cysts that develop secondary to localised bronchiolar and alveolar necrosis, which allow one way passage of air into the interstitial space. They commonly occur in immunocompetent patients and are most commonly associated with 5. aureus, followed by Staphylococcus pneumoniae infections. Although there is no dear correlation between the development of pneumatocoeles and mechanical ventilation, patients receiving mechanical ventilation have an increased risk for developing complications related to pneumatocoeles, including an increase in their size. Other than in hyperimmunoglobulin E syndrome, there is no known genetic or familial tendency for pneumatocoeles.
The majority of pneumatocoeles (more than 85%) resolve spontaneously, partially or completely over weeks to months without dinical or radiographic sequelae

61
Q

@# 103. A 3-month-old full-term infant with normal antenatal history presents with multiple seizures. On clinical examination, there is no facial asymmetry, dysmorphology or ophthalmoplegia. There is marked hypotonia of the limbs. An MRI of the brain revealed a reduction in the number of cortical sulci and shallow Sylvian fissures. What is the most likely diagnosis?
A. Holoprosencephaly
B. Lissencephaly
C. Band heterotopia
D. Hemimegalancephaly
E. Schizencephaly

A
  1. B. Lissencephaly

Lissencephaly (smooth brain) is a severe malformation of the cerebral cortex that results from impaired neuronal migration during the third and fourth months of gestation. The affected brain shows either an absence or a paucity of gyri (agyria or pachygyria, respectively).
The most common clinical manifestations include severe psychomotor retardation, developmental delay, seizures and failure to thrive. The prognosis depends on the degree of failure of cortical development. In severe cases, death occurs in infancy or early childhood. Prenatal diagnosis of an affected foetus allows appropriate counselling and optimisation of obstetric management. Abnormal cortical development is the main manifestation of lissencephaly, although other associated cranial and extracranial abnormalities may be present.

62
Q

@# 105. A 38 year old patient presents with right-sided abdominal pain. She underwent renal transplantation 2 years previously for autosomal dominant polycystic kidney disease. A CT scan of the abdomen revealed oedematous terminal ileum, caecum and ascending colon. Which of the following is the most likely diagnosis in this patient?
A. Appendicitis
B. Tuberculosis
C. Crohn’s disease
D. Typhlitis
E. Ischaemic bowel

A
  1. D. Typhlitis

Typhlitis, also known as neutropenic colitis, is a recognised acute colitis affecting the caecum ± the terminal ileum and ascending colon with a predisposition for children with leukaemia, lymphoma and patients on immunosuppressive treatment (i.e., neutropenia). The CT findings include circumferential wall thickening of the caecum, which may extend to the terminal ileum and ascending colon, pericaecal fluid and localised fat stranding. Intestinal TB may be primary or secondary to haematogenous spread from pulmonary TB, and predominantly affects the colon and ileum. Appendicitis may also produce caecal wall thickening, but it is usually asymmetric in nature and rarely extends into the terminal ileum. Backwash ileitis is a chronic complication of ulcerative colitis in which the terminal ileum is affected with a patulous ileocaecal valve, absent peristalsis and granularity of the mucosa. Pseudomembranous colitis usually has a predisposition for the distal colon but may affect the colon in its entirety.

63
Q

@# 108. A moderately large, elongated lesion is seen posteriorly in the thoracic spinal canal on the MRI of a 36-year-old apyrexial, systemically healthy man with lower limb weakness. The signal from the fluid is noted to be identical to CSF elsewhere. No history of trauma was present. What is the diagnosis?
A. Extradural arachnoid cyst
B. Extradural abscess
C. Epidural haematoma
D. Syrinx
E. Cystic meningioma

A
  1. A. Extradural arachnoid cyst

Spinal extradural arachnoid cyst (SEAC) is a rare disease and uncommon cause of compressive myelopathy. SEACs can be found in any location, although they are mostly reported to be located at the mid-thoracic to the thoracolumbar junction, commonly in a posterior position. The underlying cause is thought to be a dural defect. The cause of dural defect can be congenital or acquired. Trauma, arachnoiditis or iatrogenic cause can result in a small dural tear and subsequent CSF accumulation to develop SEACs.
MRI shows an elongated cystic mass, low on T1-weighted and high on T2-weighted images. CT myelography shows communication between the subarachnoid space and die cyst, confirming a dural tear.

64
Q

@# 110. A staging CT is performed on a patient with biopsy-proven gastric cancer. The tumour involves serosa, and enlarged nodes are present 3.5 cm from the primary tumour. No distant lesion is identified. Which one of the following is the correct TNM stage?
A. T1 N0 M0
B. Tl N1 Ml
C. T2 N1 M0
D. T2 N2 M0
E. T3 N2 MO

A
  1. D. T2 N2 M0
    Staging
    Tl = Limited to mucosa or submucosa
    T2 = Tumour involves muscle/serosa
    T3 = Tumour penetrates through serosa
    T4a = Tumour involves adjacent contagious tissues
    T4b = Invasion of adjacent structures, diaphragm, abdominal wall and so on
    N1 = Involvement of perigastric nodes within 3 cm of primary along greater or lesser curvature
    N2 = Involvement of regional nodes >3 cm from primary along branches of coeliac axis
    N3 = Para-aortic, hepatoduodenal, retropancreatic, mesenteric nodes
    Ml = Distant metastases
65
Q

@# 112. A 27-year-old woman was referred for a pelvic ultrasound by her family doctor to investigate irregular periods. Pelvic ultrasound revealed enlarged ovaries bilaterally with multiple peripheral cysts of similar size arranged like a garland around centrally increased ovarian stroma. All of the following are associated clinical features commonly observed in this clinical scenario, except:
A. Hypertension
B. Obesity
C. Insulin resistance
D. Menorrhagia
E. Hirsutism

A
  1. D. Menorrhagia
    Polycystic ovarian syndrome (PCOS) is the most common endocrine abnormality in women of reproductive age and carries with it significant health risks, including infertility, endometrial hyperplasia, diabetes (insulin resistance), obesity and cardiovascular disease (hypertension, hyperlipidaemia, coronary artery and cerebrovascular events). Patients with PCOS have hyperandrogenism, hirsutism and ovarian dysfunction (oligo- or anovulation) and present with oligo-amenorrhea.
66
Q

@# 113. An elderly woman with progressive worsening of back pain is initially investigated with plain films of the spine. Plain radiographs show Grade III collapse of L3 with Grade II collapse of at least two other mid-thoracic vertebrae. MRI suggested osteoporotic collapse as the most likely cause. Which one of the following is the expected progression of osteoporotic vertebral compression fractures as seen on MR imaging?
A. A partial return to normal fatty marrow.
B. No change.
C. The progression is unpredictable.
D. An increase in oedema and fibrovascular tissue.
E. A decrease in normal fatty marrow.

A
  1. A. A partial return to normal fatty marrow.

The signal intensity of the fractured vertebral body would appear low on T1-weighted images in the acute phase and would gradually be restored to normal intensity from the periphery to the centre of the body, as healing progresses. On T2-weighted images, the signal intensity of the fractured vertebral body would appear high, with or without some strongly lowered area in it, in the acute phase, and would be gradually restored to normal intensity with time.
Acute fracture and metastatic compression fracture can both show’ enhancement post-contrast injection. However, contrast enhancement decreases with time in benign vertebral fractures as normal marrow signal is restored.

67
Q

@# 114. A 53-year-old man with acute left leg pain in the distribution of the left L5 nerve was referred for an MRI of the lumbar spine. MRI revealed a well-defined lesion with low signal on ‘IT W images and high signal on T2W images in the lateral aspect of the spinal canal, in close relation to the L4/5 facet joint. What is the diagnosis?
A. Ganglion
B. Synovial cyst
C. Neurofibroma
D. Arachnoid cyst
E. Tarlov cyst

A
  1. B. Synovial cyst

Intraspinal synovial cysts are extradural lesions that arise from the synovial lining of the facet joints. Most cysts arc found at the L4-L5 facet joint, as this is the level where the most biomechanical spinal motion occurs.
The differential diagnosis for synovial cysts includes arachnoid cysts, perineural (Tarlov) cysts, schwannomas and migrated herniated disk fragments. The MR imaging characteristics and the neuroanatomic location of the cyst help distinguish synovial cysts from these other lesions.

Extradural arachnoid cysts are cerebrospinal fluid-filled outpouchings of the arachnoid membranes that extend through a defect in the dura mater. Two-thirds of these lesions occur in the thoracic spine; this helps to differentiate these masses from synovial cysts, which occur most frequently in the lumbar spine.

Perineural cysts can be distinguished from synovial cysts because perineural cysts are separate from the facet and are intimately associated with the nerve root. Likewise, schwannomas can be distinguished from synovial cysts on MR images by their intimate association with the nerve root. Furthermore, the propensity for schwannomas to enhance homogeneously after administration of a gadolinium containing contrast agent helps one distinguish this entity from synovial cysts, which typically demonstrate only rim enhancement. Migrated disk fragments are sometimes found dorsal to the thecal sac, which complicates their differentiation from synovial cysts. They can, however, be distinguished reliably by their relationship to the ligamentum flavum, their signal intensity characteristics and their lack of degenerative changes in the facet joint. Migrated disk fragments are typically located anterior to the ligamentum flavum, whereas synovial cysts are located dorsal to or inseparable from the ligamentum flavum. Furthermore, migrated disk fragments are usually lobulated and have lower signal intensity on T2 weighted MR images than do the spherically shaped synovial cysts.

68
Q

@# 115. A 45-year-old man presents to the A&E department with acute onset of dyspnoea. Serum biochemistry reveal an elevated creatinine consistent with established renal failure, and a chest HRCT scan shows isolated, diffuse ground-glass opacity. The most likely cause of the CT abnormality is:
A. Pulmonary alveolar proteinosis
B. Pneumocystis carinii pneumonia
C. Acute interstitial pneumonia
D. Respiratory bronchiolitis interstitial lung disease
E. Diffuse alveolar haemorrhage

A
  1. E. Diffuse alveolar haemorrhage

Anti-glomerular basement membrane antibody disease (Goodpasture syndrome) is defined by a triad of diffuse pulmonary haemorrhage, glomerulonephritis and circulating anti glomerular basement membrane antibodies. Findings at chest radiography may occasionally be normal despite the presence of diffuse pulmonary haemorrhage. Diffuse pulmonary haemorrhage can also occur in patients with systemic lupus erythematosus, typically in the context of established disease associated with extrapulmonary manifestations such as glomerulonephritis